Đến nội dung

sinh vien

sinh vien

Đăng ký: 18-04-2015
Offline Đăng nhập: 02-04-2019 - 21:50
-----

#562681 Các bài toán trên tạp chí American Mathematical Montly

Gửi bởi sinh vien trong 31-05-2015 - 17:46

11805File gửi kèm  AMM11805.pdf   34.17K   121 Số lần tải

11802File gửi kèm  AMM11802 solution1 .pdf   98.65K   130 Số lần tải-File gửi kèm  AMM 11802 solution2.pdf   119.92K   184 Số lần tải

11801File gửi kèm  AMM11801.pdf   114.15K   198 Số lần tải

11800File gửi kèm  AMM 11800.pdf   124.83K   188 Số lần tải

 




#562680 Các bài toán trên tạp chí American Mathematical Montly

Gửi bởi sinh vien trong 31-05-2015 - 17:37

       Đây là một số bài toán có lời giải chi tiết và khá rõ ràng trên Tap chí toán học Mỹ mà mình sưu tầm được :

 

11815 File gửi kèm  AMM11815.pdf   84.16K   208 Số lần tải

11814File gửi kèm  AMM11814.pdf   115.45K   153 Số lần tải

11813File gửi kèm  AMM11813.pdf   138.74K   288 Số lần tải

11812File gửi kèm  AMM11812.pdf   111.82K   156 Số lần tải

11811File gửi kèm  AMM11811.pdf   128.18K   168 Số lần tải

11810File gửi kèm  AMM11810.pdf   98.29K   167 Số lần tải

11809File gửi kèm  AMM11809.pdf   36.7K   135 Số lần tải

11808File gửi kèm  AMM11808.pdf   29.79K   137 Số lần tải

 




#562302 Các bài toán liên quan đến hàm đặc trưng của một tập

Gửi bởi sinh vien trong 29-05-2015 - 18:48

Bài toán ( Miklos Schweizer 2014 ) Cho k là một số nguyên dương và các tập$I_{1},I_{2},...,I_{k}$ là các tập con của [0,1] . Chứng minh bất đẳng thức sau:

                       $\sum_{I_{i}\bigcap I_{j}\neq \varnothing }\frac{1}{\left | I_{i}\bigcup I_{j} \right |}\geqslant k^{2}$




#562252 Chứng minh đẳng thức lượng giác sau

Gửi bởi sinh vien trong 29-05-2015 - 12:13

Bài toán.Chứng minh rằng :

   $\frac{1}{sin^{2}\frac{\pi }{4k+2}}+\frac{1}{sin^{2}\frac{3\pi }{4k+2}}+\frac{1}{sin^{2}\frac{5\pi }{4k+2}}+...+\frac{1}{sin^{2}\frac{(2k-1)\pi }{4k+2}}=2k(k+1)$




#562235 Các bài toán về số chiều và cơ sở của một không gian vector

Gửi bởi sinh vien trong 29-05-2015 - 10:29

Bài toán (ĐHSP HCM-2013) Cho $x_{1},x_{2},...,x_{n}$ là các vector khác không trong không gian vector V và $\varphi :V\rightarrow V$ là một ánh xạ tuyến tính thỏa $\varphi x_{1}=x_{2},\varphi x_{k}=x_{k}-x_{k-1},k=2,3...n$.

  Trong những trường hợp nào hệ $x_{1},x_{2},...,x_{n}$ độc lập tuyến tính.

 Lời giải. Ta sẽ chứng minh rằng hệ $\left \{ x_{1},x_{2},...,x_{n} \right \}$ độc lập tuyến tính khi n=1,2 hoặc $x_{1}-x_{2}+x_{3}\neq 0$ ( n$\geq 3$ )

 Với trường hợp n=1 , khẳng đinh hiển nhiên đúng.

 Với trường hợp n=2 . Giỉa sử $\lambda _{1}x_{1}+\lambda _{2}x_{2}=0\Rightarrow \lambda _{1}x_{2}+\lambda _{2}(x_{2}-x_{1})=0$.

  Lưu ý đến giả thiết $\lambda _{1}x_{1}+\lambda _{2}x_{2}=0$ và $x_{1},x_{2}$ là các vector khác không ta suy ra kết luận trong khẳng định.

Với trường hợp n=3. Tiếp tục giả sử $\lambda _{1}x_{1}+\lambda _{2}x_{2}+\lambda _{3}x_{3}=0\Rightarrow \lambda _{1}x_{2}+\lambda _{2}(x_{2}-x_{1})+\lambda _{3}(x_{3}-x_{2})=0$

$\Rightarrow (-\lambda _{1}-\lambda _{2})x_{1}+(\lambda _{1}-\lambda _{3})x_{2}=0\Rightarrow \lambda _{1}=-\lambda _{2}=\lambda _{3}$

 Khi đó ta viết lại giả thiết ban đầu như sau:

                 $\lambda _{1}x_{1}-\lambda _{1}x_{2}+\lambda _{1}x_{3}=0$.

Nếu $\lambda _{1}\neq 0\Rightarrow x_{1}-x_{2}+x_{3}=0$ ( trái giả thiết trong khẳng định ) 

nên $\lambda _{1}=\lambda _{2}=\lambda _{3}=0$

 Bằng một phép quy nạp đơn giản ta chứng minh được rằng :

    Nếu $x_{1}-x_{2}+x_{3}\neq 0$ thì hệ $\left \{ x_{1},x_{2},...,x_{n} \right \} (n\geq 3)$ độc lập tuyến tính.  ( Lập luận tương tự các phần trên nên không ghi ra ở đây ).

   Ta cho một ví dụ để thấy rằng tồn tại $\varphi ,x_{1},x_{2},x_{3 }$ thỏa mãn điều kiện $x_{1}-x_{2}+x_{3}= 0$ và $\varphi x_{1}=x_{2};\varphi x_{2}=x_{2}-x_{1};\varphi x_{3}=x_{3}-x_{2}$. 

   Chọn $A=\begin{bmatrix} \varepsilon &0 & 0\\ 0 &\varepsilon & 0\\ 0 & 0 & \varepsilon \end{bmatrix}$ trong đó $\varepsilon ^{2}-\varepsilon +1=0$ . Chọn $x_{1}=\begin{bmatrix} 1\\1 \\1 \end{bmatrix};x_{2}=\begin{bmatrix} \varepsilon \\\varepsilon \\\varepsilon \end{bmatrix};x_{3}=\begin{bmatrix} \varepsilon ^{2}\\\varepsilon ^{2} \\\varepsilon ^{2} \end{bmatrix}$.

Khi đó bằng cách tính toán trực tiêp ta được điều phải tìm.




#561548 tuyển chọn các bài toán tính định thức

Gửi bởi sinh vien trong 25-05-2015 - 18:59

Bài toán ( ĐH-FPT 2013 ) Tính định thức sau

         $\begin{vmatrix} x+a_{1} &a_{2} & ... &a_{n} \\a_{1} & x+a_{2} & ... & a_{n}\\ ... &... &... &... \\a_{1} & a_{2} &... & x+a_{n} \end{vmatrix}$

Cách 1 ( Biến đổi sơ  cấp )  Cộng  tất cả các cột 2 ,3,...,n vào cột đầu tiên ta thu được

                                 $\begin{vmatrix} x+a_{1}+a_{2}+...+a_{n} & a_{2} &... & a_{n}\\ x+a_{1}+a_{2}+...+a_{n}&x+a_{2} &... &a_{n} \\ ...& ....& ... &.... \\x+a_{1}+a_{2}+...+a_{n} &a_{2} &... &x+a_{n} \end{vmatrix}$

 Tiếp theo trừ hàng thứ 1 cho các hàng 2,3,...n ta được

                  $\begin{vmatrix} x+a_{1}+a_{2}+...+a_{n} & a_{2} & ... & a_{n}\\ 0 & x & ... & a_{n}\\... & ... & ... & ...\\ 0 & 0 & ... & x \end{vmatrix}$

  Đến đây dễ  thấy giá trị định thức cần tìm bằng $x^{n-1}(x+a_{1}+a_{2}+...+a_{n})$

Cách 2 ( Lý thuyết về giá trị riêng - đa thức đặc trưng )  Đặt  $P(\lambda )=det(A-\lambda I_{n})$,

trong đó $A=\begin{bmatrix} a_{1} &a_{2} & ... &a_{n} \\ a_{1} & a_{2} & ... &a_{n} \\ ...& ... & ... &... \\a_{1} & a_{2}& ... & a_{n} \end{bmatrix}$. 

  Ta thấy rank A=1 , nên 0 là một  giá trị riêng của ma trận A và có số bội bằng $n-rankA=n-1$. 

  Theo đó giá trị riêng còn lại của ma trận A  bằng $a_{1}+a_{2}+...+a_{n}$ . Lưu ý là hệ  số của $\lambda ^{n}$ trong $P(\lambda )$ là $(-1)^{n}$

nên $P(\lambda )=det(A-\lambda I_{n})(-1)^{n}\lambda ^{n-1}(\lambda +a_{1}+a_{2}+...+a_{n})$ 

 Thay $\lambda =-x$ thay thấy $det(A+xI_{n})=x^{n-1}(x+a_{1}+a_{2}+...+a_{n})$.

  Chú ý vế trái là định thức ta cần tính.

      




#561104 Chứng minh đẳng thức lượng giác sau

Gửi bởi sinh vien trong 23-05-2015 - 12:07

Bài toán . Chứng minh rằng:

                 $\sum_{k=1}^{n}\left ( tan\frac{k\pi }{2n+1} \right )^{2}=n\prod_{k=1}^{n}\left (tan\frac{k\pi }{2n+1} \right )^{2}$




#561074 tuyển chọn các bài toán tính định thức

Gửi bởi sinh vien trong 23-05-2015 - 09:12

Bài toán ( Sydney-2011) Cho m, n là hai số nguyên dương sao cho $m\geq n$ Gọi A là ma trậ vuông cấp n sao cho phần tử (i,j) bằng $C_{mj}^{i}$ . Tính det A .

    Đáp số : $detA=m^{\frac{n(n+1)}{2}}$




#560899 tuyển chọn các bài toán tính định thức

Gửi bởi sinh vien trong 22-05-2015 - 14:15

Lời giải. Ta sẽ tính định thức bằng các xác định các giá trị riêng của ma trận.

  Trước tiên để cho thuận tiện trong các trình bày ta sẽ xét một trường hợp đặc biệt khi a=0 và bc=1

kí hiệu là $D_{n}$ . Đặt $P_{n}(\lambda )=det(\lambda I_{n}-D_{n})$ .

  Áp dụng khai triển Laplace liện tiếp cho ma trận $\lambda I_{n}-D_{n}$ , lần thứ nhất theo cột thứ nhất , lần thứ hai theo hàng thứ nhất, ta thu được hệ thức

$P_{n}(\lambda )=(-1)^{1+1}\lambda P_{n-1}(\lambda )+(-1)^{2+1}(-c) (-1)^{1+1}(-b)P_{n-2}(\lambda )=\lambda P_{n-1}(\lambda )-P_{n-2}(\lambda )$

  Ta qui ước $P_{0}(\lambda )=1$, bằng tính toán trực tiếp $P_{1}(\lambda )=1$ nên với mọi $n\geq 2$ ta luôn có

                     $P_{n}(\lambda )=\lambda P_{n-1}(\lambda )-P_{n-2}(\lambda )$

 Bằng phương pháp quy nạp ta dễ dàng nhận thấy;

                                 $P_{n}(2cos\theta )=\frac{sin(n+1)\theta )}{sin\theta }, 0< \theta < \pi$

  Dễ dàng nhận thấy $P_{n}(\lambda _{k})=0$ với $\lambda _{k}=2cos\frac{k\pi }{n+1},k=1,2,...n$  mà $degP_{n}(x)=n$ . Từ đây suy ra $\left \{ 2cos\frac{k\pi }{n+1} \right \}_{k=1}^{n}$ là tất cả các giá trị riêng của $D_{n}$.

   Trong trường hợp tổng quát ta thấy $A_{n}=aI_{n}+\sqrt{bc}\overline{D_{n}}$, trong đó $A_{n}$ là ma trận cho trong đề bài còn $\overline{D_{n}}$ có các phần tử trên đường chéo chính bằng 0 và thỏa mãn các tính chất trong trường hợp riêng mà ta đã khảo sát. Theo kết quả trên ta thấy $\left \{ a+2\sqrt{bc}cos\frac{k\pi }{n+1} \right \}_{k=1}^{n}$ là các giá trị riêng của $A_{n}$

   Do đó

       $det(A_{n})=\prod_{k=1}^{n}\left ( a+2\sqrt{bc}cos\frac{k\pi }{n+1} \right )=\prod_{k=1}^{n}\left ( a-2\sqrt{bc}cos\frac{(n+1-k)\pi }{n+1} \right )=\prod_{k=1}^{n}\left ( a-2\sqrt{bc}cos\frac{k\pi }{n+1} \right )$




#560858 tuyển chọn các bài toán tính định thức

Gửi bởi sinh vien trong 22-05-2015 - 10:53

Bài toán ( Nordic-2011) Chứng minh rằng

$\begin{vmatrix} a & b & 0 & 0 & 0 & ... & 0 & 0 &0 \\ c &a &b & 0 & 0 & ... & 0 &0 &0 \\0 & c & a & b &0 & ... & 0 & 0 &0 \\... & ... & ... &... &... &... &... &... &... \\ 0 & 0 & 0 & 0 &0 & ... & a &b &0 \\0 & 0 & 0 & 0 & 0 & ... & c & a &b \\ 0 & 0 & 0 & 0 & 0 &... &0 &c & a \end{vmatrix}_{n\times n}=\prod_{k=1}^{n}\left ( a-2\sqrt{bc}cos\frac{k\pi }{n+1} \right )$




#560844 tuyển chọn các bài toán tính định thức

Gửi bởi sinh vien trong 22-05-2015 - 07:51

Bài toán ( Sydney-2005). Tính định thức của ma trận vuông A cấp n trong đó

                                                             $a_{ij}=\frac{(2i+2j-2)!}{2^{2i+2j-2}(i+j-1)!}$

Lời giải. Ta chứng minh kết quả sau:

  Cho  $\alpha \in \mathbb{R}$ , giả sử $C=(c_{ij})_{i,j=1}^{n}$ thỏa $c_{i,j+1}=(i+j-\alpha )c_{ij}$

 với mọi $1\leq i\leq n,1\leq j\leq n-1$ thì $detC=\prod_{i=1}^{n}(i-1)!c_{i,1}$




#560738 tuyển chọn các bài toán tính định thức

Gửi bởi sinh vien trong 21-05-2015 - 17:51

Bài toán ( Sydney-2009 ) Cho $A_{n}$ là ma trận vuông cấp n trong đó phần tử (i,j) bằng 1 nếu $n\leq i+j\leq n+1$ và bằng 0 trong các trường hợp còn lại.

 Tìm các giá trị riêng của $A_{n}$




#560719 tuyển chọn các bài toán tính định thức

Gửi bởi sinh vien trong 21-05-2015 - 16:37

Bài toán ( Sydney-2013). Tính định thức của ma trận vuông cấp n trong đó phần tử (i,j) bằng i nếu $i\neq j$ và bằng i+1 nếu i=j

Lời giải. Gọi $M_{n}$ là ma trận được định nghĩa, $I_{n}$ là ma trận đơn vị cấp n.

Khi đó ta thấy từ định nghĩa thì ma trận $M_{n}-I_{n}$ có hàng thứ i là (i,i,...,i) nên có hạng bằng 1 và định thức bằng 0 .

 Suy ra 1 là một giá trị riêng của ma trận $M_{n}$ có số bội bằng n-1.

Ta thấy vết của ma trận $M_{n}$ bằng $\sum_{i=1}^{n}(i+1)=\frac{n^{2}+3n}{2}$ nên giá trị riêng còn lại của ma trận $M_{n}$ bằng $\frac{n^{2}+3n}{2}-(n-1)=\frac{n^{2}+n+2}{2}$

  Do đó : $detM_{n}=1^{n-1}\times\frac{n^{2}+n+2}{2}=\frac{n^{2}+n+2}{2}$




#560292 Tuyển tập đề thi Annual Vojtech Jarnik 1991-2015

Gửi bởi sinh vien trong 19-05-2015 - 10:06

Lưu ý với các bạn là có một số bài không thấy ghi lời giải

 1991 File gửi kèm  j01solutions.pdf   111.92K   192 Số lần tải               1998 File gửi kèm  j08solutions.pdf   175.1K   177 Số lần tải

 1992File gửi kèm  j02solutions.pdf   114.65K   147 Số lần tải                1999File gửi kèm  j09solutions.pdf   147.21K   176 Số lần tải

 1993File gửi kèm  j03solutions.pdf   130.89K   180 Số lần tải                 2000File gửi kèm  j10solutions.pdf   164.37K   185 Số lần tải

 1994File gửi kèm  j04solutions.pdf   124.13K   130 Số lần tải                 2001File gửi kèm  j11solutions.pdf   180.96K   146 Số lần tải

1995File gửi kèm  j05solutions.pdf   133.48K   141 Số lần tải                  2002File gửi kèm  j12solutions.pdf   146.28K   289 Số lần tải

 1996File gửi kèm  j06solutions.pdf   117.64K   124 Số lần tải                 2003File gửi kèm  j13solutions.pdf   132.82K   170 Số lần tải

 1997File gửi kèm  j07solutions.pdf   176.82K   149 Số lần tải                 2004File gửi kèm  j14solutions.pdf   258.96K   150 Số lần tải

                                    2005 File gửi kèm  j15solutions.pdf   156.88K   170 Số lần tải

                                    2006File gửi kèm  j16solutions.pdf   162.29K   144 Số lần tải

         2007File gửi kèm  j17solutions1.pdf   85.79K   246 Số lần tảiFile gửi kèm  j17solutions2.pdf   77.4K   173 Số lần tải

        2008File gửi kèm  j18solutions1.pdf   86.36K   146 Số lần tảiFile gửi kèm  j18solutions2.pdf   91.7K   171 Số lần tải

        2009File gửi kèm  j19solutions1.pdf   81.52K   137 Số lần tảiFile gửi kèm  j19solutions2.pdf   87.67K   134 Số lần tải

        2010File gửi kèm  j20solutions1.pdf   87.35K   166 Số lần tảiFile gửi kèm  j20solutions2.pdf   90.56K   149 Số lần tải

        2011File gửi kèm  j21solutions1.pdf   148.38K   139 Số lần tảiFile gửi kèm  j21solutions2.pdf   149.88K   145 Số lần tải

        2012File gửi kèm  j22solutions2.pdf   185.63K   141 Số lần tảiFile gửi kèm  j22solutions1.pdf   211.59K   137 Số lần tải

        2013File gửi kèm  j23solutions1.pdf   136.86K   144 Số lần tảiFile gửi kèm  j23solutions2.pdf   156.03K   148 Số lần tải

        2014File gửi kèm  j24solutions1.pdf   133.64K   148 Số lần tảiFile gửi kèm  j24solutions2.pdf   155.63K   137 Số lần tải

        2015File gửi kèm  j25solutions1.pdf   147.43K   209 Số lần tảiFile gửi kèm  j25solutions2.pdf   157.17K   173 Số lần tải




#560145 tuyển chọn các bài toán tính định thức

Gửi bởi sinh vien trong 18-05-2015 - 13:15

Bài toán. Cho ma trận vuông A cấp n , với $a_{ij}=(i,j)$, trong đó (i,j) là ước chung lớn nhất của i và j

Tính det A